LSAT and Law School Admissions Forum

Get expert LSAT preparation and law school admissions advice from PowerScore Test Preparation.

 nrpandolfo
  • Posts: 33
  • Joined: Feb 04, 2018
|
#45628
Can you explain this one please?
 James Finch
PowerScore Staff
  • PowerScore Staff
  • Posts: 943
  • Joined: Sep 06, 2017
|
#45648
Hi NR,

This question is answered by an inference that should be understood during the setup to the game. The rule that we can't have two watercolors on the same wall means that we also can't have two oils on the same wall; if we did, we wouldn't have enough oils to avoid putting two watercolors together. So each wall will have one oil and one watercolor. Combined with the rule that F and I cannot be together means that IO will have either a GW or HW hanging on the same wall. And because another rule tells us that GW will hang with FO, we know by inference that IO must then hang with HW. Finally, our last rule tells us where IO goes, which means we also know where HW goes: the upper position on wall 4.

Hope this clears things up!

Get the most out of your LSAT Prep Plus subscription.

Analyze and track your performance with our Testing and Analytics Package.